Jump to content

MPMin

Senior Members
  • Posts

    262
  • Joined

  • Last visited

Everything posted by MPMin

  1. I’ll try to explain in stages, let me know which part you disagree with: Wire A and B are both physically connected to the same craft, you’re right there; but that fact alone doesn’t mean they are exerting a force on each other. Wire A and B will only produce a force on each other only when a current is present in both wires at the same time, then the forces will cancel each other out and the craft won’t move as have you described many times but that’s not the situation I’m describing in my design. If there is a current in only one of the wires, no force will occurs between the wires. This point isn’t that important at this stage but it might help you understand the next point. This is the crucial point: If you pulse a current through one of the wires, the emp detaches from the wire. Even though the emp originated from one of the wires on the craft, the detached emp is no longer part of the craft so when it interacts with the other wire, the other is effectively acting on a force that isn’t part of the craft anymore thus momentum of the whole craft can occur by the other wire acting on the detached emp.
  2. I get what you are saying but your example is not the same kind of pushing, the emp will only cause moment if a current is presenting the wire.
  3. I think if one of the wires has no current there will be no force in any direction between the wires
  4. I get your example but it needs to relate to my design to have relevance, Ive said my system is an open system, it needs energy in and it loses energy out, my design is not in a box and I’m not saying you are saying that but if you are going to say my design is violation of newtons law tell me how? The argument that the wires are on the same rig doesn’t contravene newtons laws because you aren’t considering the detachment of the emp from wire A. Once the emp leaves wire A it is no longer part of the rig so when the detached emp from wire A interacts with wire B, wire B is effectively interacting with an outside field.
  5. Don’t take this as an attack but the reason I don’t accept your argument/s is because they are circular and contradictory and inconclusive, see above - how can you expect me to draw a conclusion from inconstant information? And ‘i admit’ that if you put any propulsion system in a box it wont go anywhere, case in point, this point is inconclusive. I too have provided references and shown my workings so i think the way forward is to break down my design into individual phases like the strokes of an engine to see where the problem is Are you saying that if one of the wires in parallel carries 10 amps and the other has no current that the force of attraction or repulsion (depending on direction of current) will be equal between the two wires? Are you referring to the diagram that is using a continuous currents to show the magnetic field lines and resulting force lines? With reference to my design, just considering the first cycle, are you saying the radiation from A is blocked by wire B thus the unblocked or unbalanced radiation from A escaping out the back is what’s pushing the craft to the left? While you think about that, does the radiation from B have any effect on that cycle or is it just A? If you don’t fully understand my design how can you possibly conclude its not consistent with Newton’s laws? To break down my design in to phases, lets consider the first phase of creating the emp from wire A. If a very short pulse of current is pulsed in the wire A an emp should emerge. The purpose of pulsing the current is to detach the emp from wire A. Does anyone disagree that once the current stops in wire A, the emp detaches from wire A thus no longer having any connection or effect to wire A. This leaves wire A free to move inside the emp ring. Is there anything wrong with this concept?
  6. Despite all the angst i do actually appreciate everyone’s meaningful contribution. I still have not seen a counter argument that accurately takes the full scope of my design into account in their rebuttal. I still think most of you don’t fully understand my design, so while your rebuttals make sense to you, your rebuttals do not completely correlate to my design. I’d appreciate it even more if you all went out on a limb and really tried to understand my design instead of assuming I’m just trying to defy any Newton’s laws. If your Prima face impression of my design is just I’m trying to defy newtons law you’ll most likely not try to understand what I’m saying. My design doesn’t defy Newton’s laws: In the first phase, a current is pulse in Wire A, the emp from wire A cancels out its own effect on wire A as the magnetic field emanates outward from the wire equally in all directions - no law broken here. The next phase is to stop the current in wire A producing a short emp between the wires (instead of a magnetic field that extends from wire A to wire B as is shown in all worked examples because they only consider continuous currents instead of pulsed currents). The emp emanates outward from wire A and leaves wire A behind in all direction. As the emp travelling away from wire A it no longer effects wire A because its detached, there is a gap between the emp and wire A - no law broken here the next phase is when the magnetic pulse from A arrives at wire B, wire B has a brief current pulsed through it to create an emp at wire B, this causes a force on wire B but not on wire A thus moving the entire craft to the left as per my animation. no law has been broken here either. Most of the rebuttals revert back to my design defying newtons laws but i have not seen an argument that demonstrates this referencing my design accurately
  7. Yes of course, and of all the threads on this site, you just happen to post your meaningless observation in this particular thread...
  8. Are you suggesting that the moment you stop the current in wire A that the Magnetic field remains intact to wire A?
  9. Imagine my disappointment at seeing more snide remarks instead of seeing a logical argument countering my last post!
  10. I did apply this law, the emp from wire A cancels out its own effect on wire A as the magnetic field emanates outward from the wire equally in all directions - no law broken here. The magnetic field travelling out from wire A no longer effects wire A because its detached - no law broken here When the magnetic pulse from A arrives at wire B it causes momentum in wire B because magnetic fields are known to move wires, that’s how electric motors work and no law has been broken here either. Where has Newtons law been violated?
  11. You have stressed this but I haven’t. You are still assuming the forces are continuous when they are not. Have you seen my animation? If this principal is wrong explain how Instead of saying its wrong for reasons I’m not stating
  12. Funny that, it was my friend who convinced me to come here because the scientific community would embrace something new. Nice!
  13. Was this you showing me my error in my calculations or you showing your lack of understanding of physics? See this is what it looks like when you actually reference something someone said and ask for an explanation on it.
  14. Without making direct references to my design with tangible explanations this is just an empty statement
  15. I have provided evidence with references to the basis of what makes my design work the way it does. With regards to my references; what is the force on the two wires with a continuous current then? Radiation escaping in both directions?
  16. Either you don’t get what I’m saying or are choosing not to. I am not proposing a continuous currents to produce two continuous counteracting forces. When you can explain what’s wrong with my design in the frame of reference that I have provided or as per my animation at least then your argument will apply, until then, you are effectively talking about something else.
  17. What if this is the first time this has been thought of, if so how can I show you another example. When Edison proposed the light bulb I’m glad you weren’t there to say ‘show me someone else has made a light bulb if you want me to believe such a thing could work’ if it makes you feel better you can consider the wires are mounted outside the craft, it makes no difference if the force occurs inside or outside the craft, the point is the system generates a momentary emp then the wire pulses a current to push or pull against that emp you show me where it has been documented that this would not work OK. Lets try this. Rather than surrounding it completely, lets leave an opening on one side:   But if we put the opening on the other side, then the thrust is in the opposite direction.  This does not relate to my proposal or the animation I provided, you are still fixated on radiation escaping rather than magnetic fields interacting with current carrying wires as Ive referenced. You should also notice that my animation shows how the force is generated to the left in both phases instead of left and right as you have shown above.
  18. All the other force vectors do not interact with the craft, the force vectors that do not interact with the opposing wires are equal from both wires and cancel each other out leaving only the disparity between the forces that do interact to be considered.
  19. You are right and I’ve provided as much proof as is available considering this hasn’t been tried or conceived before to my knowledge. I have provided well documented evidence that wires with a continuous current will attract or repel each other or in other words exert a force on each other, my innovation is to cause a detachment in the magnetic field from on of the wires so that they emitted magnetic field briefly exerts a force on the other wire while it has a brief current, I provided an animation to illustrate this. The proof needs to come from actually trying it.
  20. It would help if you fully understood what ive been saying, while you misrepresent what I’m saying your arguments aren’t applicable
  21. Because the energy being put into the system from the solar panels would have no where to go, its the same as putting a normal rocket in a box, your analogy doesn’t provide any insight here
  22. You’ll have to do better than that if you want to maintain the pretence that you are sticking to the science as you yourself demand from others
  23. No one said anything about trying random stuff and don’t try telling anyone here that before every innovation was tried the results were correctly anticipated every time. On the contrary, look at the direction of the forces, this is exactly why its the force on the wires and not radiation escaping. Look at the example I provided, slow it down and check the force interaction against the references I provided, they match.
  24. Check the direction of current in A and the direction of the magnetic field from B which produce a force in the same direction as wire B, if you check my references you will see this interaction would produce a force to the left. I’m saying the force of a magnetic field interacting with a write carrying a current is far greater than the radiation from a laser.
×
×
  • Create New...

Important Information

We have placed cookies on your device to help make this website better. You can adjust your cookie settings, otherwise we'll assume you're okay to continue.